Menu Close

Category: Others

7-3-with-out-calculator-

Question Number 111643 by Dwaipayan Shikari last updated on 04/Sep/20 $$\left(\frac{\mathrm{7}}{\mathrm{3}}\right)!\left({with}\:{out}\:{calculator}\right) \\ $$ Answered by mathdave last updated on 04/Sep/20 $${solution}\: \\ $$$${but}\:{x}!=\Gamma\left({x}+\mathrm{1}\right) \\ $$$${let}\left(\frac{\mathrm{7}}{\mathrm{3}}\right)!=\Gamma\left(\frac{\mathrm{10}}{\mathrm{3}}\right)=\mathrm{2}.\mathrm{8871}…

A-number-is-said-to-be-right-prime-if-despite-dropping-the-left-most-digits-successively-Number-continues-to-be-a-prime-number-For-example-223-is-a-right-prime-because-despite-dropping-2-fr

Question Number 46045 by Tawa1 last updated on 20/Oct/18 $$\mathrm{A}\:\mathrm{number}\:\mathrm{is}\:\mathrm{said}\:\mathrm{to}\:\mathrm{be}\:''\mathrm{right}\:\mathrm{prime}''\:\mathrm{if}\:\mathrm{despite}\:\mathrm{dropping}\:\mathrm{the}\:\mathrm{left}\:\mathrm{most} \\ $$$$\mathrm{digits}\:\mathrm{successively},\:\:\mathrm{Number}\:\mathrm{continues}\:\mathrm{to}\:\mathrm{be}\:\mathrm{a}\:\mathrm{prime}\:\mathrm{number}.\:\: \\ $$$$\mathrm{For}\:\mathrm{example}:\:\:\mathrm{223}\:\mathrm{is}\:\mathrm{a}\:\mathrm{right}\:\mathrm{prime}\:\mathrm{because}\:\mathrm{despite}\:\mathrm{dropping}\:\mathrm{2}\:\mathrm{from}\:\mathrm{left} \\ $$$$\mathrm{most}\:\mathrm{part},\:\mathrm{we}\:\mathrm{obtain}\:\:\mathrm{23}\:\mathrm{as}\:\mathrm{the}\:\mathrm{prime}\:\mathrm{number}.\:\mathrm{Next},\:\mathrm{even}\:\mathrm{after}\:\mathrm{dropping}\: \\ $$$$\mathrm{2}\:\mathrm{from}\:\mathrm{left},\:\:\mathrm{3}\:\mathrm{is}\:\mathrm{a}\:\mathrm{prime}. \\ $$$$\:\:\:\:\:\mathrm{How}\:\mathrm{many}\:\mathrm{two}\:\mathrm{digit}\:\mathrm{numbers}\:\mathrm{are}\:\mathrm{right}\:\mathrm{prime}\:? \\ $$ Answered by MJS…

Let-2-3-5-6-7-10-11-be-increasing-sequence-of-positive-integers-that-are-neither-the-square-nor-cube-of-an-integer-Find-the-2016th-term-of-this-sequence-

Question Number 111536 by Aina Samuel Temidayo last updated on 04/Sep/20 $$\mathrm{Let}\:\mathrm{2},\mathrm{3},\mathrm{5},\mathrm{6},\mathrm{7},\mathrm{10},\mathrm{11},…\:\mathrm{be}\:\mathrm{increasing} \\ $$$$\mathrm{sequence}\:\mathrm{of}\:\mathrm{positive}\:\mathrm{integers}\:\mathrm{that}\:\mathrm{are} \\ $$$$\mathrm{neither}\:\mathrm{the}\:\mathrm{square}\:\mathrm{nor}\:\mathrm{cube}\:\mathrm{of}\:\mathrm{an} \\ $$$$\mathrm{integer}.\:\mathrm{Find}\:\mathrm{the}\:\mathrm{2016th}\:\mathrm{term}\:\mathrm{of}\:\mathrm{this} \\ $$$$\mathrm{sequence}. \\ $$ Commented by Rasheed.Sindhi…

Let-a-b-c-be-real-numbers-such-that-a-b-c-0-Prove-that-a-2-b-2-c-2-4-ab-bc-ca-3-27-0-

Question Number 177068 by LOSER last updated on 30/Sep/22 $${Let}\:{a},{b},{c}\:{be}\:{real}\:{numbers}\:{such}\:{that}: \\ $$$${a}+{b}+{c}=\mathrm{0}.\:{Prove}\:{that}: \\ $$$$\frac{{a}^{\mathrm{2}} {b}^{\mathrm{2}} {c}^{\mathrm{2}} }{\mathrm{4}}+\frac{\left({ab}+{bc}+{ca}\right)^{\mathrm{3}} }{\mathrm{27}}\leqslant\mathrm{0} \\ $$ Answered by Frix last updated…

Question-45992

Question Number 45992 by sandeepkeshari0797@gmail.com last updated on 19/Oct/18 Answered by tanmay.chaudhury50@gmail.com last updated on 19/Oct/18 $$\mathrm{2}+\mathrm{3}×\mathrm{1}=\mathrm{5} \\ $$$$\mathrm{2}+\mathrm{5}×\mathrm{2}=\mathrm{12} \\ $$$$\mathrm{3}+\mathrm{6}×\mathrm{3}=\mathrm{21} \\ $$$$\mathrm{8}+\mathrm{11}×\mathrm{4}=\mathrm{52}\:\:\:{i}\:{think}\:{so}… \\ $$$$…

find-u-n-0-e-n-x-cos-nx-dx-and-v-n-0-e-n-x-sin-nx-dx-2-find-nature-of-u-n-v-n-and-u-n-v-n-

Question Number 45976 by maxmathsup by imad last updated on 19/Oct/18 $${find}\:{u}_{{n}} =\:\int_{\mathrm{0}} ^{\infty} \:{e}^{−{n}\left[{x}\right]} {cos}\left({nx}\right){dx}\:{and}\:{v}_{{n}} =\int_{\mathrm{0}} ^{\infty} \:{e}^{{n}\left[{x}\right]} {sin}\left({nx}\right){dx} \\ $$$$\left.\mathrm{2}\right)\:{find}\:{nature}\:{of}\:\Sigma\:{u}_{{n}} {v}_{{n}} \:\:{and}\:\Sigma\:\frac{{u}_{{n}} }{{v}_{{n}}…

Show-that-1-2-2-2-3-2-n-n-1-2-1-2-2-2-2-3-n-2-n-1-3n-5-3n-1-

Question Number 45932 by Tawa1 last updated on 18/Oct/18 $$\mathrm{Show}\:\mathrm{that}:\:\:\:\:\frac{\mathrm{1}.\mathrm{2}^{\mathrm{2}} \:+\:\mathrm{2}.\mathrm{3}^{\mathrm{2}} \:+\:…\:+\:\mathrm{n}\left(\mathrm{n}\:+\:\mathrm{1}\right)^{\mathrm{2}} }{\mathrm{1}^{\mathrm{2}} .\mathrm{2}\:+\:\mathrm{2}^{\mathrm{2}} .\mathrm{3}\:+\:…\:+\:\mathrm{n}^{\mathrm{2}} \left(\mathrm{n}\:+\:\mathrm{1}\right)}\:\:=\:\:\frac{\mathrm{3n}\:+\:\mathrm{5}}{\mathrm{3n}\:+\:\mathrm{1}} \\ $$ Commented by math khazana by abdo last…